Позитивне топологічне впорядкування, візьміть 3


20

Припустимо, маємо матрицю n від n. Чи можливо впорядкувати його рядки та стовпці таким чином, щоб ми отримали верхньо-трикутну матрицю?

Це питання мотивоване цією проблемою: Позитивне топологічне впорядкування

Початкова проблема рішення є принаймні такою ж важкою, як ця, тому результат повноти NP також вирішив би це.

Редагувати: Ласло Вег та Андрас Френк звернули мою увагу на еквівалентну проблему, яку задав Gunter Rote: http://lemon.cs.elte.hu/egres/open/Graphs_extendable_to_a_uniquely_matchable_bipartite_graph

Редагувати: Зведення до початкової проблеми полягає в наступному. Припустимо, що DAG має лише два рівні, вони відповідатимуть рядкам і стовпцям матриці. Також у нас є один єдиний вузол вагою +1. Усі інші на нижньому рівні мають вагу -1, а на верхньому - +1.


Як ви зводите це до початкової проблеми? До речі, ця проблема сама по собі виглядає цікавою.
Цуйосі Іто

Ви шукаєте одну перестановку, яка буде застосована до рядків і стовпців, або дві окремі перестановки? Я здогадуюсь двох, оскільки лише з однією проблемою здається еквівалентною топологічному роду.
Воррен Шуді

Розглядаючи його як двосторонній графік (як у посиланні elte), вони дають необхідну умову, щоб він не мав підграфа, складеного з копій K2, C4, C6, C8 тощо. Ще одна необхідна умова - послідовність ступенів обох частини переважають (1, 2, 3, ..., n) --- Я думаю, що це сильніше, ніж інші умови, засновані на кліці в посиланні.
daveagp

Відповіді:


12

Проблема виявилася неповною. Детальніше ви можете прочитати тут і тут . Короткий підсумок:

Дасгупта, Цзян, Каннан, Лі та Сведєк: зменшення, пов'язане з зменшенням проблеми: з урахуванням двопартійного графіка G та цілого числа k, вирішить, чи має G індукований підграф на 2k вузлах, на які можна поширити бути однозначно сумісними. Stéphane Vialette помітив, що це зводить до двопартійної унікальної версії цієї проблеми, якщо ми додамо nk ізольовані вузли до обох класів.


Дякуємо за посилання на EGRES. Мені дуже подобаються відкриті проблеми, особливо ті, що стосуються (ідеального) узгодження.
Мохаммед Аль-Туркстані

Які ще є якісні сайти з відкритими проблемами (пов'язані з обчислювальною складністю)?
Мохаммед Аль-Туркстані

@turkistany, я не знаю жодних інших, я думаю, це також більше стосується дослідження операцій / теорії графіків.
domotorp

3

Увага: Це часткова відповідь, що ґрунтується на здогадах та чутках! Тоді як загальна проблема Девіда Еппштейна не є повною, але ця проблема є в П.

(AB,E)|A|=|B|=n

  • він не повинен містити 2 ідеальних відповідностей,
  • (1,2,...,n)

Поки що я не зміг знайти жодного прикладу, коли графік відповідає цим умовам, але не може бути UPMX. У такому випадку, можливо, їх достатньо. Це можна довести за допомогою наступного алгоритму:

  1. якщо графік має> 1 ідеальні відповідність, поверніть "не UPMX"
  2. якщо графік не відповідає умові ступеня, поверніть "не UPMX"
  3. якщо на графіку = 1 ідеальна відповідність, поверніть "UPMX"
  4. інакше, можливо, ми можемо показати, що це UPMX. Можливо, такий алгоритм міг би довести це:
    • поки граф має ребра,(n+12)2
    • знайти якийсь новий край e, додавання якого не створює ідеального узгодження та не порушує умови ступеня; додати е до графіка
  5. тепер графік має ребра і не має ідеального узгодження і задовольняє умові ступеня. Я думаю, що це не надто складно показати, що це UPMX, отже, так і був оригінальний графік.(n+12)1

Ви можете охарактеризувати, які нові ребра створили б ідеальну відповідність, використовуючи теорему Холла, і не важко охарактеризувати, які нові ребра порушувалимуть ступінь. На жаль, навіть якщо це правда, що край потрібного типу завжди існує, я не зміг це довести.


Непоганий підхід, мені цікаво, чи правда це.
domotorp

3

Цей документ, одержуючи трикутну матрицю незалежними перестановками рядків-стовпців Фертином, Русу та Віалеттою, показує, що задача є повною NP для двійкових квадратних матриць.


Це дуже прикро, що вони також довели той самий результат незалежно від нас, я думаю, ми повинні були спілкуватися краще. У будь-якому разі я їм надішлю електронний лист.
domotorp

@domotorp Таку ж проблему задали і в MathOverflow, і найкраща відповідь була в тому, що вона знаходиться в "NP-кінцівці". mathoverflow.net/questions/191963/…
Мохаммед Аль-

-1

Проблема неповна, але де алгоритм її вирішення? У мене є один алгоритм, який працює на багатьох прикладах, але я не можу демонструвати його роботу весь час.


1
Чи можете ви охарактеризувати цікавий клас графіків, на яких правильний ваш алгоритм?
РБ
Використовуючи наш веб-сайт, ви визнаєте, що прочитали та зрозуміли наші Політику щодо файлів cookie та Політику конфіденційності.
Licensed under cc by-sa 3.0 with attribution required.